LSAT and Law School Admissions Forum

Get expert LSAT preparation and law school admissions advice from PowerScore Test Preparation.

 Administrator
PowerScore Staff
  • PowerScore Staff
  • Posts: 8916
  • Joined: Feb 02, 2011
|
#40616
Complete Question Explanation
(The complete setup for this game can be found here: lsat/viewtopic.php?t=13252)

The correct answer choice is (A)

As discussed during the setup, S cannot have been written third as it ultimately leads to a violation of the fourth rule. Thus, answer choice (A) cannot occur and must be the correct answer.

Note also that in question #1, a full, viable solution to the game is produced, and H was written third in that solution. Thus, for this question, if you did not have the S Not Law on the third position in your initial setup, by referring to the answer in question #1 you could have at least eliminated answer choice (D) as a contender.

Get the most out of your LSAT Prep Plus subscription.

Analyze and track your performance with our Testing and Analytics Package.